LSAT and Law School Admissions Forum

Get expert LSAT preparation and law school admissions advice from PowerScore Test Preparation.

 Administrator
PowerScore Staff
  • PowerScore Staff
  • Posts: 8916
  • Joined: Feb 02, 2011
|
#26507
Complete Question Explanation

Question #18: Must Be True. The correct answer choice is (B).

The stimulus contains a fact set. Scientists receive the most recognition when overthrowing conventional wisdom, and apparently global warming is part of that wisdom (it's "widely accepted"). Some scientists are skeptical of it, but very few find it to be unlikely - despite the apparent motivation to overthrow conventional wisdom.

The stem asks us to determine what must be true. Proceed by the process of elimination: any answer choice that does not pass the Fact Test must be immediately eliminated.

Answer choice (A) is incorrect because we have no information about the accepted moral standards for scientific debate - our evidence instead speaks of the motivations of scientists in their research. Thus, answer choice (A) cannot be proven with the information presented in the stimulus.

Answer choice (B) is the correct answer choice. We are told that scientists have a motivation to overthrow conventional wisdom, and we are also told that the idea of global warming is widely accepted - making it, effectively, conventional wisdom. We can prove, then, that most scientists have substantial motive to disprove it.

Answer choice (C) is incorrect, because the stimulus does not allow us to determine whether the evidence conclusively proves the global warming hypothesis.

Answer choice (D) is incorrect, because it falls entirely outside the scope of the fact set. It's entirely possible that the scientists in question have actually offered an alternative hypothesis.

Answer choice (E) is incorrect, because it contains an exaggeration: we cannot prove that research in global warming is primarily driven by a desire for recognition in the scientific community.
 sgrimsdale
  • Posts: 14
  • Joined: Sep 01, 2014
|
#18842
Hi,

I am a little confused by why B is a better answer than A. Can someone help me out?

Thanks!

Sarah
 David Boyle
PowerScore Staff
  • PowerScore Staff
  • Posts: 836
  • Joined: Jun 07, 2013
|
#18852
sgrimsdale wrote:Hi,

I am a little confused by why B is a better answer than A. Can someone help me out?

Thanks!

Sarah
Hello Sarah,

Answer A is tempting, maybe, but it doesn't really overgeneralize. If parents hide an innocent child, that could indeed be thought of as "obstructing police work".
Answer B is better in that the "widely recognized" part of it counters the "widely accepted" in the stimulus, and because the other countervailing principles help override the parents' "duty" to hide their child, so that there isn't, after all, a presumption that it's o.k. to obstruct police work.

Hope this helps,
David
 Zierra28
  • Posts: 17
  • Joined: Aug 12, 2015
|
#19858
I chose A for #18 regarding moral dilemmas. Can you help me understand why it's not as good an option as the correct answer, B? Thanks!

Also Can you break down the conditional reasoning in #25 of that section as well, please?
 Clay Cooper
PowerScore Staff
  • PowerScore Staff
  • Posts: 241
  • Joined: Jul 03, 2015
|
#19865
Hi Zierra,

Thanks for your questions.

As to the first, answer choice A is incorrect because we have no information about the accepted moral standards for scientific debate - our evidence instead speaks of the motivations of scientists in their experimental research. Thus, we will not be able to prove that A is true.

B, by contrast, we can prove. We are told that scientists have motivation to overthrow conventional wisdom, and we are also told that the idea of global warming is widely accepted - making it, effectively, conventional wisdom. We can prove, then, that most scientists have good motivation to try to disprove it.

As for #25, the only conditional reasoning I see in this stimulus is that, since academy support stifled innovation in sculpture, AS --> xI. This relationship suggests that C is correct, since the far greater number of unsponsored paintings than unsponsored sculptures means that painters likely were more often free of the need for academy support, and thus they were not as often subject to its stifling effect on innovation.

Hope that helps!
 LSAT2018
  • Posts: 242
  • Joined: Jan 10, 2018
|
#45362
I immediately eliminated answers (A) and (b) because of the 'most scientists' part. Given parts of the stimulus such as 'some scientists are skeptical of the widely accepted predictions of global warming' and 'hundreds of researchers striving to make breakthroughs in climatology,' isn't 'most scientists' unsupported?
 lsatnoobie
  • Posts: 52
  • Joined: Sep 18, 2017
|
#45630
Hi Powerscore!

How can we assume that “most researchers” have motive to discredit the global warming hypothesis when the stimulus says “it is accordingly unsurprising that SOME scientists are skeptical of the widely accepted predictions of global warming.”
 Daniel Stern
PowerScore Staff
  • PowerScore Staff
  • Posts: 81
  • Joined: Feb 07, 2018
|
#45832
The stimulus tells us that, "Nothing brings more recognition than overturning conventional wisdom," and further that predictions about global warming are "widely accepted," meaning that these predictions are the conventional wisdom.

So, since it brings recognition -- indeed, more recognition than anything else -- to overturn conventional wisdom, and global warming is conventional wisdom, we have support for answer choice B, that most researchers have a powerful incentive to find evidence against global warming.

The fact that only some scientists are doing this doesn't change the fact that many may have an incentive to do so; perhaps they are merely not acting on that incentive. It is this very situation that the author finds "surprising" in the last sentence.

Good luck in your studies,
Dan
 student987
  • Posts: 28
  • Joined: Apr 09, 2018
|
#50102
Hello! When I first solved this, I didn't think Answer (A) was talking about moral standards. Can someone explain how they made the jump? Should I translate "the accepted standards of x" to "moral standards," whenever it comes up? Thank you!

Clay Cooper wrote:Hi Zierra,

Thanks for your questions.

As to the first, answer choice A is incorrect because we have no information about the accepted moral standards for scientific debate - our evidence instead speaks of the motivations of scientists in their experimental research. Thus, we will not be able to prove that A is true.

B, by contrast, we can prove. We are told that scientists have motivation to overthrow conventional wisdom, and we are also told that the idea of global warming is widely accepted - making it, effectively, conventional wisdom. We can prove, then, that most scientists have good motivation to try to disprove it.

As for #25, the only conditional reasoning I see in this stimulus is that, since academy support stifled innovation in sculpture, AS --> xI. This relationship suggests that C is correct, since the far greater number of unsponsored paintings than unsponsored sculptures means that painters likely were more often free of the need for academy support, and thus they were not as often subject to its stifling effect on innovation.

Hope that helps!
 Brook Miscoski
PowerScore Staff
  • PowerScore Staff
  • Posts: 418
  • Joined: Sep 13, 2018
|
#58891
987,

Feel free to remove the word "moral" from the explanation. Answer Choice (A) denotes the standards of debate, which was not discussed. Some people might connote "moral" from a standard of conduct, others might connote "ethical," and others might not connote at all. Regardless of how you interpreted "standards of debate," it wasn't discussed.

Get the most out of your LSAT Prep Plus subscription.

Analyze and track your performance with our Testing and Analytics Package.